LSAT and Law School Admissions Forum

Get expert LSAT preparation and law school admissions advice from PowerScore Test Preparation.

User avatar
 Dave Killoran
PowerScore Staff
  • PowerScore Staff
  • Posts: 5853
  • Joined: Mar 25, 2011
|
#46603
Complete Question Explanation
(The complete setup for this game can be found here: lsat/viewtopic.php?t=17159)

The correct answer choice is (B)

The condition in the question stem enacts the second rule: 2S :arrow: 1R :arrow: 3T. This leads to the following initial setup:
D00_Game_#3_#18_diagram 1.png
However, more information can be derived. When M is selected, then from the fourth rule W is selected. When W is selected, from the third rule Z cannot be selected. And, from the third rule, when W is selected, H is not selected, and so only F or G remain for the ruby selection:
D00_Game_#3_#18_diagram 2.png
Thus, answer choice (B) could be true and is correct.
You do not have the required permissions to view the files attached to this post.
 srcline@noctrl.edu
  • Posts: 243
  • Joined: Oct 16, 2015
|
#21337
Hello

I am having difficulty with question 18. Here are my initial inferences:

W :dblline: H
W :dblline: Z
M :dblline: H

So for question 18 is this the right hypothetical for the reason B is correct?

J M F X W Y (out) H Z G K

Thank you
Sarah
 Lucas Moreau
PowerScore Staff
  • PowerScore Staff
  • Posts: 216
  • Joined: Dec 13, 2012
|
#21342
Hello, srcline,

Your first two inferences are robust, but I think your third one could be a little more specific. It's true that M :dblline: H, but it is also true that M :dblline: Z for the same reason. I would style it like this instead:

W :dblline: H
W :dblline: Z
M :arrow: W :arrow: Not!H and Not!Z

As far as number 18, if J and M are the only sapphires selected, that triggers Rule 2, and exactly one ruby must be selected. Furthermore, M means we have to have W, and cannot have H or Z. That knocks out answer choices C and E immediately. :) Answer choice D goes out due to the fact that K cannot be picked without taking all three sapphires, and we can likewise eliminate answer choice A due to that giving us two rubies. By process of elimination, answer choice B is the only possibility.

Hope that helps,
Lucas Moreau

Get the most out of your LSAT Prep Plus subscription.

Analyze and track your performance with our Testing and Analytics Package.